Difference between revisions of "2018 AMC 10B Problems/Problem 5"

(Video Solution)
(Solution 5)
(3 intermediate revisions by 2 users not shown)
Line 5: Line 5:
 
<math>\textbf{(A)} \text{ 128} \qquad \textbf{(B)} \text{ 192} \qquad \textbf{(C)} \text{ 224} \qquad \textbf{(D)} \text{ 240} \qquad \textbf{(E)} \text{ 256}</math>
 
<math>\textbf{(A)} \text{ 128} \qquad \textbf{(B)} \text{ 192} \qquad \textbf{(C)} \text{ 224} \qquad \textbf{(D)} \text{ 240} \qquad \textbf{(E)} \text{ 256}</math>
  
==Solution 1==
+
==Solution 1 - Complementary Counting==
  
We use complementary counting, or Total- what we don't want = what we want. There are a total of <math>2^8</math> ways to create subsets (consider including or excluding each number) and there are a total of <math>2^4</math> subsets only containing composite numbers. Therefore, there are <math>2^8-2^4=240</math> total ways to have at least one prime in a subset.
+
We use complementary counting, or  
 +
 
 +
<math>\text{total - what we don't want = what we want}</math>.  
 +
 
 +
There are a total of <math>2^8</math> ways to create subsets (consider including or excluding each number) and there are a total of <math>2^4</math> subsets only containing composite numbers (the composite numbers are <math>4, 6, 8, 9</math>). Therefore, there are <math>2^8-2^4=240</math> total ways to have at least one prime in a subset.
  
 
==Solution 2 (Using Answer Choices)==
 
==Solution 2 (Using Answer Choices)==
Line 36: Line 40:
  
 
-goldenn
 
-goldenn
 +
 +
==Solution 5==
 +
We multiply the number of possibilities of the set having prime numbers and the set having composites.
 +
 +
The possibilities of primes are <math>2^4-1=15</math> (As there is one solution not containing any primes)
 +
 +
The possibilities of the set containing composites are <math>2^4=16</math> (There can be a set with no composites)
 +
 +
Multiplying this we get <math>15 \cdot 16 = \boxed{\textbf{(D) }240}</math>.
 +
 +
-middletonkids
  
 
==Video Solution==
 
==Video Solution==

Revision as of 15:12, 10 August 2020

Problem

How many subsets of $\{2,3,4,5,6,7,8,9\}$ contain at least one prime number?

$\textbf{(A)} \text{ 128} \qquad \textbf{(B)} \text{ 192} \qquad \textbf{(C)} \text{ 224} \qquad \textbf{(D)} \text{ 240} \qquad \textbf{(E)} \text{ 256}$

Solution 1 - Complementary Counting

We use complementary counting, or

$\text{total - what we don't want = what we want}$.

There are a total of $2^8$ ways to create subsets (consider including or excluding each number) and there are a total of $2^4$ subsets only containing composite numbers (the composite numbers are $4, 6, 8, 9$). Therefore, there are $2^8-2^4=240$ total ways to have at least one prime in a subset.

Solution 2 (Using Answer Choices)

Well, there are 4 composite numbers, and you can list them in a 1 number format, a 2 number, 3 number, and a 4 number format. Now, we can use combinations.

$\binom{4}{1} + \binom{4}{2} + \binom{4}{3} + \binom{4}{4} = 15$. Using the answer choices, the only multiple of 15 is $\boxed{\textbf{(D) }240}$

By: K6511

Solution 3

Subsets of $\{2,3,4,5,6,7,8,9\}$ include a single digit up to all eight numbers. Therefore, we must add the combinations of all possible subsets and subtract from each of the subsets formed by the composite numbers.

Hence:

$\binom{8}{1} - \binom{4}{1} + \binom{8}{2} - \binom{4}{2} + \binom{8}{3} - \binom{4}{3} + \binom{8}{4} - 1 + \binom{8}{5} + \binom{8}{6} + \binom{8}{7} + 1 = \boxed{\textbf{(D) }240}$

By: pradyrajasai


Solution 4

Total subsets is $(2^8) = 256$ Using complementary counting and finding the sets with composite numbers: only 4,6,8 and 9 are composite. Each one can be either in the set or out: $2^4$ = 16 $256-16=240$ $\boxed{\textbf{(D) }240}$

-goldenn

Solution 5

We multiply the number of possibilities of the set having prime numbers and the set having composites.

The possibilities of primes are $2^4-1=15$ (As there is one solution not containing any primes)

The possibilities of the set containing composites are $2^4=16$ (There can be a set with no composites)

Multiplying this we get $15 \cdot 16 = \boxed{\textbf{(D) }240}$.

-middletonkids

Video Solution

https://youtu.be/6Bt4I23JL1s

~savannahsolver

See Also

2018 AMC 10B (ProblemsAnswer KeyResources)
Preceded by
Problem 4
Followed by
Problem 6
1 2 3 4 5 6 7 8 9 10 11 12 13 14 15 16 17 18 19 20 21 22 23 24 25
All AMC 10 Problems and Solutions

The problems on this page are copyrighted by the Mathematical Association of America's American Mathematics Competitions. AMC logo.png